Difference between revisions of "2023 WSMO Speed Round Problems/Problem 7"
(Created page with "==Problem== Let <math>e, a, j</math> be real numbers such that <math>e + a + j = 1</math> and <math>e\geq -\frac{1}{3}</math>, <math>a\geq 1</math> and <math>j\geq-\frac{5}{3...") |
|||
Line 4: | Line 4: | ||
==Solution== | ==Solution== | ||
+ | From the Cauchy-Schwarz inequality, we have | ||
+ | <cmath>\begin{align*} | ||
+ | \left(\sqrt{3e+1}+\sqrt{3a+3}+\sqrt{3j+5}\right)^2&\le\left((3e+1)+(3a+3)+(3j+5)\right)\left(1+1+1\right) | ||
+ | &\le\left(3(e+a+j)+9\right)(3) = (3(1)+9)(3)=36\implies\\ | ||
+ | \sqrt{3e+1}+\sqrt{3a+3}+\sqrt{3j+5}\le\sqrt{36}=\boxed{6}. | ||
+ | \end{align*}</cmath> | ||
+ | |||
+ | ~pinkpig |
Revision as of 11:32, 12 September 2025
Problem
Let be real numbers such that
and
,
and
. Find the maximum value of
Solution
From the Cauchy-Schwarz inequality, we have
~pinkpig